LSAT and Law School Admissions Forum

Get expert LSAT preparation and law school admissions advice from PowerScore Test Preparation.

User avatar
 Dave Killoran
PowerScore Staff
  • PowerScore Staff
  • Posts: 5850
  • Joined: Mar 25, 2011
|
#72675
Parallel, SN. The correct answer choice is (D).

The stimulus begins with with background information regarding a new shopping center, which the Shopkeeper states won't be a spot for their store relocation. The reason is contained in the next two sentences, which begin with a conditional relationship:

  • ..... ..... ..... ..... ..... High Visibility
    Store ..... :arrow: ..... ..... ..... +
    ..... ..... ..... ..... ..... Good Growth Potential
This information is paired with the premise that only two spots are being considered for the center: Maple Street, which lacks visibility, and West Avenue, which lacks growth potential. These two pieces of information each deny a separate necessary condition, which enacts a contrapositive of the above relationship, namely that:
  • High Visibility
    ..... or ..... ..... ..... ..... :arrow: ..... Store
    Good Growth Potential
Thus, since Maple Street lack visibility, the store can't be located there, and since West Avenue lacks growth, the store can't be located there. Consequently, based on the given conditional relationship and the additional information, the Shopkeeper's conclusion is sound.


Answer choice (D): This is the correct answer choice. The initial sentence can be diagrammed as followed:

  • ..... ..... ..... ..... ..... Small Venue
    Sell Out ..... :arrow: ..... ..... ..... +
    ..... ..... ..... ..... ..... Downtown

There are then two locations under consideration, Jensen Arena and Pembroke Hall. But each has a flaw that fails to meet one of the necessary conditions above, and in each case that enacts a contrapositive that leads to the conclusion the concert will not sell out.
 annrachweila
  • Posts: 8
  • Joined: Sep 12, 2019
|
#72747
Hi,
I chose C initially but ended up switching to D. I was wondering if my reasoning for the elimination of C is correct:
C states that Finch won't travel unless he can get inexpensive plane tickets OR stay with a friend, which seems like 2 separate conditions. However, the stimulus says that the store needs to be in a high-visibility site WITH good growth potential, making it sound like a single condition. D has a similar pattern as the stimulus ("small venue in the downtown area"). Am I right to eliminate C this way? Thank you!
User avatar
 Dave Killoran
PowerScore Staff
  • PowerScore Staff
  • Posts: 5850
  • Joined: Mar 25, 2011
|
#72753
Hi Ann,

Basically yes, although I'd argue that whereas (C) has an "or" in the necessary condition, both the stimulus and (D) have an "and." You called it a single condition (and I'd say you understood what was happening so it wasn't an issue) whereas I would just split that into its two component parts and show it as "and" (as shown above).

Thanks!
 RAB
  • Posts: 18
  • Joined: Oct 01, 2020
|
#84222
Dear Dave,

Is the reason (A) is incorrect due to the fact that it is not about two location choices. Appiah would rent if there is no increase and pets are allowed. So the contrapositive would be if rent increases or no pets Appiah would not rent.

Thoughts welcome.
RAB
 Rachael Wilkenfeld
PowerScore Staff
  • PowerScore Staff
  • Posts: 1358
  • Joined: Dec 15, 2011
|
#84269
Hi Rab,

You are correct that the stimulus has an additional factor of locations. It starts by saying that a store needs two key criteria (high visibility and good growth potential). There are two locations under consideration, and one lacks the first requirement, while the second lacks the other requirement. Therefore, shopkeeper won't be moving to either potential location.

Answer choice (A) lacks the idea of the two potential options. It jumps straight from her two conditions to the fact that her apartment won't meet either condition. That's different structurally than the stimulus.

Hope that helps!
 RAB
  • Posts: 18
  • Joined: Oct 01, 2020
|
#84308
Thank you Rachel.
 lsatep2024
  • Posts: 4
  • Joined: Dec 30, 2023
|
#104720
Can (A) also be eliminated because it's taking two different conditions rather than two qualities/conditions about a single event? That was what stood out to me the most when I was going between that and my final choice (D)

Thank you!
 Robert Carroll
PowerScore Staff
  • PowerScore Staff
  • Posts: 1783
  • Joined: Dec 06, 2013
|
#104931
lsatep2024,

Answer choice (A) is actually two reasons a single thing won't happen, rather than two different reasons two different options won't be satisfactory, like the stimulus is. It structurally does not match.

Robert Carroll

Get the most out of your LSAT Prep Plus subscription.

Analyze and track your performance with our Testing and Analytics Package.